Your-Doctor
Multiple Choice Questions (MCQ)


Quiz Categories Click to expand

Category: Q&A Medicine--->Nephrology
Page: 4

Question 16# Print Question

A 28-year-old IV drug user presents to the hospital with shortness of breath. The murmur of tricuspid regurgitation is heard on cardiac auscultation, and a chest x-ray shows multiple opacities in both lungs. An echocardiogram confirms endocarditis, and the patient is started on empiric vancomycin and gentamicin while blood cultures are pending. Two days later, the following results are seen on her laboratory tests.

  • Blood urea nitrogen   28 mg/dL
  • Creatinine   1.8 mg/dL (baseline 0.9 mg/dL)
  • Urinalysis   Pigmented granular casts

Which of the following should be done next?

A. IV fluids
B. Temporary hemodialysis
C. Stop gentamicin
D. Stop vancomycin
E. Immediate surgery of the infected valve


Question 17# Print Question

A 76-year-old man is hospitalized for pneumonia and was started on empiric antibiotics. He developed a rash in response to the antibiotics, and the symptoms were somewhat relieved with diphenhydramine. Several days after this, the patient developed oliguria. His laboratory values are shown below.

  • Sodium   138 mEq/L
  • Potassium   4.8 mEq/L
  • Chloride   108 mEq/L
  • Bicarbonate   20 mEq/L
  • Blood urea nitrogen   18 mg/dL
  • Creatinine   1.6 mg/dL (baseline 1.1 mg/dL)
  • Glucose   114 mg/dL 

A urinalysis is unremarkable. A renal ultrasound shows bilateral dilation of the renal pelvis and calyces.

Which of the following is the most likely anatomic site of obstruction in this patient?

A. Renal arteries
B. Ureters
C. Renal tubules
D. Bladder
E. Renal veins


Question 18# Print Question

A 63-year-old man complains of persistent abdominal pain for several weeks. In addition, he endorses an unintentional weight loss of 10 kg over the past few months. His medical history is significant for hypertension and diabetes. He has a 45 pack-year smoking history and drinks alcohol moderately. His hemoglobin is found to be 18.8 g/dL, and his urinalysis shows 23 RBCs per high power field. A CT scan is performed and shows a large mass within the left kidney.

Besides erythrocytosis, what is another potential complication of this disease?

A. Limbic encephalitis
B. Muscle weakness and double vision
C. Hypercalcemia
D. Hyponatremia


Question 19# Print Question

A 28-year-old man presents to his physician with complaints of fatigue and muscle cramps that have developed slowly over the past few months. In addition, he says that he often needs to get up in the middle of the night to urinate. He has no significant past medical history or family history, does not take any medications or supplements, eats a regular diet, and does not smoke, drink alcohol, or use illicit drugs. He denies any recent illnesses, vomiting, or diarrhea. His temperature is 36.8°C, blood pressure is 102/68 mmHg, heart rate is 82 beats per minute, and respiratory rate is 10 breaths per minute. His laboratory values and urine studies are shown below.

  • Sodium   136 mEq/L
  • Potassium   2.6 mEq/L
  • Chloride   93 mEq/L
  • Bicarbonate   34 mEq/L
  • Blood urea nitrogen   9 mg/dL
  • Creatinine   0.8 mg/dL
  • Glucose   150 mg/dL (non-fasting)
  • Magnesium   0.6 mEq/L

Arterial blood gas

  • pH   7.50
  • PaO2   95 mmHg
  • PaCO2   48 mmHg

Urine studies show high levels of potassium and chloride, and low levels of calcium.

Which of the following conditions would produce similar findings to this patient’s disease?

A. Chronic diarrhea
B. Adrenal aldosterone-secreting tumor
C. Chronic chlorthalidone use
D. Chronic furosemide use


Question 20# Print Question

A 53-year-old man with a history of diabetes and diabetic nephropathy is undergoing workup for a suspected diagnosis of coronary artery disease. His exercise stress test results place him in the high risk category, and he elects to undergo further workup with coronary angiography. The procedure was uneventful, and 2 days later he has laboratory tests drawn that are significant for a creatinine of 1.9 mg/dL (baseline 1.3 mg/dL). The serum leukocyte count, as well as the differential, is normal. A urinalysis shows pigmented granular casts with no RBCs or white blood cells (WBCs), and a FENa is calculated at 0.85%. The patient denies any flank pain or dysuria, and his vitals, physical examination, and urine output are normal.

Which of the following is the most likely diagnosis?

A. Reaction to a contrast agent
B. Renal atheroemboli
C. Prerenal acute kidney injury
D. Acute interstitial nephritis
E. Acute tubular necrosis from renal ischemia




Category: Q&A Medicine--->Nephrology
Page: 4 of 8